You are on page 1of 20

EEET2197 Mechatronics & Control REVISION EXERCISE 1 (2012)

APR 2013
EEET2197 MECHANTRONICS AND CONTROL - Apr 2013
1 1. Draw the diagram of a closed-loop feedback control system, where the plant transfer function
is Gp(s), the controller transfer function is C(s) and the transfer function for the sensor is H(s). On
the diagram, mark set-point signal R(s), control signal U(s) and output signal Y (s), feedback
error E(s).
2. Find the following closed-loop transfer functions:
Y (s)/R(s); E(s)/R(s); U(s)/R(s)
1S

C(s) Gp(s)

H(s)

The controller receives the error signal as its input and determines in real time the corrective
action to be applied to the plant/process as a function of the input error signal. The output
signal of the controller is the input signal to the plant/process being controlled. This signal is
called the manipulated variable. For example, in the context of speed control of an automobile,
the accelerator pedal position is the manipulated variable. Typical PI Controller function
t
K
u (t ) = K C (r (t ) y (t )) + C (r ( ) y ( ))d
Ti s 0
KC
U ( s ) = K C ( R( s ) Y ( s )) + ( R ( s ) Y ( s))
Ti s
1S Y ( s) C ( s )G p ( s ) E ( s) 1 U (s) C ( s)
= = =
R ( s ) 1 + C ( s )G p ( s ) H ( s ) R( s) 1 + C ( s)G p ( s) H ( s) R( s) 1 + C ( s)G p ( s) H ( s)
2 s +1
Consider the transfer function G ( s ) =
( s + 10)(s 2 + 2 s + 2)
1. Find the poles and zero of the transfer function.
2. Mark the locations of the poles and zero on the complex plane.
3. Is the transfer function stable? Why?
4. Find the second order approximate transfer function for G(s). Why do you think it is a good
approximation?
2S Poles: ( s + 10)( s 2 + 2 s + 2) = 0
s=-10 or s = -1j
Zeros: -s+1=0 s=1 POLES X ZEROS

X O

2S Yes, the transfer function is stable because all poles are the left half s-plane
2S s +1 s +1 0.05(1 s )
G ( s) = =
( s + 10)( s 2 + 2 s + 2) 2 x10(0.1s + 1)(0.5s 2 + s + 1) (0.5s 2 + s + 1)
2S The time constant t=0.1 is 5 times less than the second order time constant, so we can neglect
it, we keep the steady-state gain the same and keep the unstable zero.
3 0.01
Suppose that the transfer function model of a water tank is given by G p ( s ) = and a
s + 0. 1

-1-
EEET2197 Mechatronics & Control REVISION EXERCISE 1 (2012)

K c ( s + 0.1)
controller is chosen to have the structure C ( s ) =
s
1. What Kc value do we have to use in order to have a closed-loop system with a closed-loop
pole at 0.5? Explain your answer.
2. Supposing that the reference signal is a step signal with amplitude of 0.1, show that the
steady-state error Lim ( r (t ) y (t )) = 0 as long as Kc is positive.
t
3S The closed-loop characteristics equation polynomial
0.01 K c ( s + 0.1) 0.01K c
1 + G p ( s )C ( s ) = 1 + = 0 1 + G p ( s )C ( s ) = 1 + =0
s + 0.1 s s
The closed-loop pole is s + Kc 0.01=0 pole = -0.01 Kc
-0.5= -0.01 Kc Kc=5
We need to put the closed-loop pole at -0.5. We need to solve the polynomial equation:
s+0.01Kc=s+0.5
3S The transfer function of the error signal E(s) is :
R(s) R(s) sR ( s )
E ( s) = = =
1 + G ( s )C ( s ) 1 + 0.01K c / s s + 0.01K c
Kc > 0, then the closed-loop pole is stable, we can apply the final value theorem
s 0. 1
Lim ( r (t ) y (t )) = Lim( s )=0
t s 0 s + 0.01K c s
4 Y (s) s+K
Assume that a closed-loop control system has the transfer function = 3
R( s) s + 2s 2 + 4s + K
Determine the minimum and maximum values of K such that the closed-loop system is stable
by using Routh-Hurwitz criterion.
4S s 3 + 2 s 2 + 4 s + K K > 0 and K > 8,
S 3
1 4 s=j j 3 2 2 + 4 j + K = 0
S2 2 K =2 and K = 8
S1 (K-8)/2 0
S0 K
5 K
The characteristic equation of a feedback control system is 1 +
( s + 5)( s 2)
1. Find the parameter K when the damping factor of the closed-loop system is 0.7.
2. Sketch the root-locus of the closed-loop system for K.
5S The closed-loop characteristic equation is
( s + 5)( s 2) + K = 0
s 2 + 3s 10 + K = 0
s 2 + 2n s + n = 0
2

n =(K-10) and 2n=3 -1.5


-5 2
(K-10) = 3/(2) = 3 / (2 x 0.7) K = (3/1.4)2+10
S=[-3[9-4(K-10)]/2=-3/2[49-4k]/2

DEC 2012
EEET2197 DEC 2012 EXAM
1a Draw a general block diagram of a closed-loop control system.
Above each arrow in the block diagram, write down the generic name of the signal that the
arrow represents.
Also write down the generic name of each block in the block diagram.
1b A position control system is governed by the following equations:
d 2 y (t ) dy (t )
2
+3 + 2 y (t ) = (t ) e(t ) = r (t ) y(t )
dt dt

-2-
EEET2197 Mechatronics & Control REVISION EXERCISE 1 (2012)

d (t )
(t ) + 0.01 = 0.25v(t ) v(t ) = 8e(t )
dt
The variables involved are as follows:
r(t) = desired position (reference signal) y(t) = actual position (controlled signal)
e(t) = amplifier input voltage (error signal) v(t) = amplifier output voltage
(t) = motor shaft angular position
Draw a block diagram of the system, identifying the component parts and their
transmittances (or transfer functions), and determine the system transfer function Y(s)/R(s).
1aS Disturbance
Signal

Comparator Controller Controlled system


Reference Error Manipulated Controlled
Signal Analogue/Digital Signal Signal Analogue/Digital Signal Variable Machine, Plant, or Variable
Processing Processing Process

Feedback Transducer
Feedback
Signal Appropriate Sensing Device for the particular controlled signal
+ Signal Processing Electronics

1bS

Y (s ) 200
T (s ) = = 3
R(s ) s + 103s + 302s + 400
2

2a A system is shown in Figure 2a. Determine the transfer functions Y(s)/R(s) and Y(s)/Td(s). If
both r(t) and td(t) remain constant over time and have values 1.25 and 0.5 respectively,
determine the steady state value yss of y(t).
Td(s)
-
R(s)+ + Y(s)
_

Figure 2a
2b The block diagram of a high-performance servo drive that incorporates inner feedback loops and
feed forward control is shown in Figure 2b. Determine the transfer function Y(s)/R(s) and Y(s)/Td(s).

-3-
EEET2197 Mechatronics & Control REVISION EXERCISE 1 (2012)

Figure 2b
2aS 10 2

Y (s ) (s + 1) (s + 4) 20(50s + 1)
GR (s ) = = =
R (s ) 1 + 10

2

1 (s + 1)(s + 4)(50s + 1) + 20
(s + 1) (s + 4) (50s + 1)
2
Y (s ) (s + 4 ) 2(s + 1)(50s + 1)
GTd (s ) = = =
Td (s ) 1+
10

2

1 (s + 1)(s + 4)(50s + 1) + 20
(s + 1) (s + 4) (50s + 1)
yss = GR (0) rss + GTd (0 ) td ,ss =
(20 1) 1.25 (2 11) 0.5 = 1.0
(1 4 1 + 20) (1 4 1 + 20)
2bS 200 K i k
K p Kv t2
Y (s ) s Js 2 (s + 200 + 200Ki ) Js
GR (s ) = = 1+ +
R (s ) K p k t K p K v
200K i k s
1 + K p Kv t 2 1 +
(s + 200 + 200K i ) Js K p

=
( )
200 K i Js 2 + kt K v s + kt K p K v
(s + 200 + 200K )Js + 200k K K (s + K )
i
2
t v i p

1
Y (s ) Js 2
GTd (s ) = =
Td (s ) 200K i k s
1 + K p Kv t 2 1 +
(s + 200 + 200K i ) Js K p

=
(s + 200 + 200K i )
(s + 200 + 200K i )Js 2 + 200kt K v K i (s + K p )
3 A unity feedback system is shown in Figure 3.
R(s)+ Y(s)

.
3.a What is the type number of the system? What is the steady state error to (i) unit step input,
and (ii) unit ramp input?
3.b Determine the closed-loop transfer function Y(s)/R(s). Hence determine the damping ratio,
natural frequency, and settling time (for 2% overshoot). Does the system zero, s = -52, have
any significant effect on the system dynamics?
3.c Write down the expression for the output Y(s) to a unit step input R(s) = 1/s. Express Y(s)
-4-
EEET2197 Mechatronics & Control REVISION EXERCISE 1 (2012)

as a partial fraction, and by inverse Laplace transformation, determine the expression for
the time function y(t).
3.d How many approximate number of oscillations would be present in the transient response
y(t) before the system settles down to within 2% of the final value.
3.e Sketch the transient response y(t) and mark the key values.
3aS System is Type 1.
Steady state error to unit step input is zero.
Steady state error to a unit ramp input = 1/52
3bS 2(s + 52 )
Y (s ) s (s + 2 ) 2(s + 52) 2(s + 52 )
= = = 2
R (s ) 1 + 2 ( s + 52 ) s(s + 2 ) + 2(s + 52 ) s + 4 s + 104
s (s + 2 )
2(s + 52 ) 2(s + 52 )
= =
(s + 2) + 10 (s + 2 + j10)(s + 2 j10)
2 2

Natural frequency = 104 10.2 rad/s


4 2
Damping ratio = 0.196
2 104 10.2
4
Settling time = = 2.0 seconds
2
The zero at s = -52 has no effect as it is more than 10 times further into the negative half of
the s-plane when compared to the complex pole pair.
3cS 2(s + 52 ) 1 A Bs + C
Y (s ) = 2 = + 2
s + 4 s + 104 s s (
s + 4 s + 104 )
( )
or 2(s + 52) = A s + 4s + 104 + (Bs + C )s
2

Choosing s = 0, we have 2(0 + 52 ) = A(0 + 0 + 104 ) , or A = 1.


Choosing s = -1, we have 2( 1 + 52) = 1(1 4 + 104) + B C , or B-C = 1.
Choosing s = +1, we have 2(1 + 52) = 1(1 + 4 + 104) + B + C , or B+C = -3.
B = -1, and C = -2.
1
Hence, Y (s ) = 2
s+2 1
=
(s + 2 )
( )
s s + 4 s + 104 s (s + 2 )2 + 10 2
Inverse Laplace transform, y (t ) = 1 e 2t cos(10t )
3dS Settling time = 2 seconds; Period of oscillation =2/10 seconds.
Hence the number of oscillations before settlement = 20/2, or approximately three
oscillations.
3eS

-5-
EEET2197 Mechatronics & Control REVISION EXERCISE 1 (2012)

4a Briefly explain what root locus plot is, and its role in the design of control systems.
4b For the system shown in Figure 4, determine the characteristic equation. Arrange the
characteristic equation in the standard form for plotting root locus. Hence determine the
open-loop poles, open-loop zeros, and the gain K (in terms of a).

Figure 4
4c Sketch the root locus and comment on the effect of increasing the value of K (i.e., decreasing
the value of a) on the stability and transient response to a step input of the closed-loop
system.
4d What values of a would result in complex closed-loop poles?
4e What would you choose as the optimum value of a, and why?
4aS Root locus is the plot of all the poles of the closed-loop transfer function in the complex
s-plane as one of the parameters appearing in the characteristic equation (say, K) is given
different positive values in the range zero to infinity.
Here we assume that numerical values of all the remaining parameters of the characteristic
equation are either known in advance or assigned a fixed set of values.
From the root locus plot it is very easy to see how changes in the value of the parameter
affect the closed-loop system poles and hence the stability and transient response.
By inspecting the root locus plot, we may be able to select an optimum value for the
parameter K.
4bS
Characteristic equation is 1 +
1

(s + 9) = 0 .
11(s + 5) (as + 1)
11(s + 5)(as + 1) + (s + 9 ) = 0 11as 2 + 55as + 11s + 55 + s + 9 = 0
16 16
s + s +
12 s + 64 12 3
1+ K
3
1+ =0 1+ =0 =0
11as(s + 5) 11a s (s + 5) s (s + 5 )
12
Hence K = ; two open-loop poles at 0 and -5; one open-loop zero at -16/3.
11a
4cS Segments of the real axis that form part of the root locus are (i) between the two open-loop
poles 0 and -5, and (ii) between the open-loop zero -16/3 and -.
Only one asymptote: At angle = -1800 to real axis (i.e., the negative real axis).
16
s + (2s + 5) s(s + 5)
s (s + 5 )
=
dK 3
K = Break points are given by = 0.
16 ds 16
2
s + s +
3 3
32 80 2 32 80
2 s 2 + s + 5s + s 5s = 0 s2 + s + =0
3 3 3 3
32 32 2 4 3 80 16 4 20
Roots are: s = s= or s= and s = 4
6 3 3 3
20
s= lies between the open-loop zero -16/3 and - and is therefore a break-in point
3
s = 4 lies between the two open-loop poles 0 and -5, and is therefore a break-away point

-6-
EEET2197 Mechatronics & Control REVISION EXERCISE 1 (2012)

-20/3
-16/3 -5 0
-4

4dS s (s + 5 )
K =
16
s +
3
Values of K at the two break points, s = -4 and s = -20/3, are:
20 20
+ 5

( 4)( 4 + 5) = 3 and
3 3 = 25
16 20 16 3
4 + +
3 3 3
12 12
Corresponding values of a are: and .
11 3 11
25
3
Complex closed-loop poles occur when a lies between 4/11 and 36/275.
4eS It is desirable to have a damping ratio as close to 0.7 as possible. The closest value of
damping ratio we can achieve corresponds to the tangent line from the origin to the circle.
For this case,
2 2 2 2
16 4 4 1 4
s s+5


3 3
3 3
(16 1) 1 (16 1)
K= = = 3 3 =5
16 4 4
s+
3 3 3
12 12
Corresponding value of a = = .
11 5 55
4
15
15
Minimum possible damping ratio = 3 =
16 4
3
Using MATLAB

-7-
EEET2197 Mechatronics & Control REVISION EXERCISE 1 (2012)

Selected point = -5.1161 + j1.3277


K = 5.2361
5a Answer ANY THREE of the following FOUR parts (a, b, c and d).
In the context of a closed-loop control system, what is meant by the open-loop frequency
response.
Discuss the application of the open-loop frequency response plot in the design of control
systems.
5b In industrial process control terminology, what is meant be percentage proportional band
(%PB)?
A temperature controller has a 60% proportional band. Its input range is 500C to 1000C and
its output range is 4 mA to 20 mA. Assuming that the set point is 750C (50% of
measurement), plot a graph to show the relationship between the percentage control effort
and the percentage deviation.
What is the percentage control effort when the measurement is 800C? What is the
corresponding value of the controller output in mA
5c Draw a block diagram of a typical digital closed-loop control system. Clearly label the
various signals in the block diagram, indicating whether they are discrete-time, or
continuous-time signals. Also sketch typical time plots of these signals.
5d Define mechatronics. Explain the difference between a mechatronic system and a traditional
control system.
5aS
r(t) = r0 cos(t) + y(t) = y0 cos(t+)
GC(s) GP(s)
_

H(s)

Closed loop frequency response is the plot of y0/r0 and versus the frequency .
To obtain the open loop frequency response, the feedback signal is disconnected at the
summing junction as shown below:

-8-
EEET2197 Mechatronics & Control REVISION EXERCISE 1 (2012)

r(t) = r0 cos(t) +
GC(s) GP(s)

z(t) = z0 cos(t+*)
H(s)

Open loop frequency response is the plot of z0/r0 and * versus the frequency .
Methods based on the open loop frequency response can be used to investigate the
stability of feedback control systems, and to design appropriate compensators.
5bS In industrial controllers the gain of the controller is described in terms of its proportional
band (PB) or its percentage proportional band (%PB).
Proportional band represents fractional deviation (or fractional change in measured value)
that will generate 100% change in control effort.
With a narrower proportional band, changes in measured value outside that proportional
band will have no effect in the controller output, i.e. it will be saturated at 0% or 100%.

5cS

5dS Mechatronics is the interdisciplinary field of engineering dealing with the design of products
whose function relies on the integration of mechanical and electronic components
coordinated by a control architecture.
In control engineering, the optimisation problem is formulated as: given a process to be
controlled, and given the cost function (performance index), find optimal controller
parameters such that the cost function is minimised.
Mechatronic design on the contrary requires that not only the controller, but also the process
itself, as well as the sensors and actuators are optimised simultaneously.

-9-
EEET2197 Mechatronics & Control REVISION EXERCISE 1 (2012)

DEC 2011
EEET2197 DEC 2011 EXAM
Q1 Using the Laplace Transform Table, find the inverse transforms (i.e., g(t)) of the following Laplace 6%
a Transform functions:
2 2 2 (s + 1)
(i) G(s ) = (ii) G(s ) = (iii) G(s ) =
(s + 1) (s + 1) (s + 1)2 + 4 2
1b Comment on how their magnitudes change with the passage of time in each case. 4%
1c Draw rough sketches (graphs) to show how g(t) varies with time t in each case in the time interval 8%
0 t 2 seconds. What is significant about this time interval?
1d How would the graphs differ if the common factor (s + 1) in each of the above Laplace Transform 2%
functions is replaced with (s - 2)?
1aS 2
(i) G(s ) = g (t ) = 2e t 2%
(s + 1)
2 1%
(ii) G(s ) = t
(s + 1) g (t ) = 2e 3%
2 (s + 1)
(iii) G(s ) = g (t ) = 2e t cos(2t )
(s + 1)2 + 4 2
1bS (i) Exponentially decaying waveform with an initial magnitude of 2. 1%
(ii) Mirror image about the time axis of the above. 1%
(iii) Exponentially decaying waveform with an initial magnitude of 2 modulated by a cosine 2%
function of period 1 second. 6%
1cS The interval, 0 t 2 , 6%
corresponds to the
first two complete
cycles of the
waveform 2%

1dS The magnitude, instead of decreasing exponentially, would increase, with the passage of time, 2%
exponentially at twice the rate from the initial value of 2
2a A system is shown in 10%
Figure 2a. Determine the
transfer function Y(s)/R(s).

Figure 2a

-10-
EEET2197 Mechatronics & Control REVISION EXERCISE 1 (2012)

2b A mechanical system is depicted in Figure 2b. The 10%


input signal is the force f(t) directly applied to the
Spring
mass m, and the output signal is the vertical Force
Constant
displacement of the mass. Determine the transfer f(t) or F(s)
k
function Y(s)/F(s) in terms of the parameters m, k Mass
and b. e m = 2 kg and k = 200 N/m. What are the Mass Displacement
values of static gain and natural frequency? What y(t) or Y(s)
should be the value of b to achieve a damping ratio
m
of 0.5?

Figure 2b
2aS

4%

3%

1
Y (s ) s(s + 5)( s + 10) 1 1
= = 3 = 3
R( s ) ( s + 108s + 996) s + 15s + 50s + s + 108s + 996 s + 16s + 158s + 996
2 2 2 2 3%
1+
s(s + 5)( s + 10)
2bS d2y dy 3%
Applying Newtons Law to the mass, m 2
= f (t ) ky b
dt dt


Laplace transforming (assuming zero initial conditions) and rearranging,
1%
Ms2 Y(s)+bsY(s)+kY(s)=F(s) Substituting the given numerical values
Y ( s) 1 0.005
G ( s) = = 2 = 2 2%
F ( s) 2 s + bs + 200 s / 10 + b / 200 s + 1
2

Static gain = 0.005 m/N 1%


Natural frequency=10 rad/s 1%
Damping ratio is given by, b /200=2x0.5/10 Hence, the required value of b is 20 N/(rad/s). 2%
Q3 An arrangement for the open loop speed control of a permanent magnet DC motor is shown
schematically in the block diagram of Figure 3.

M(s) Power PM DC (s)


Amplifier Motor
Motor Figure 3
The transfer function relating the speed of the motor to the input voltage signal is given by,
(s ) 60
= (rad/s)/V
M (s ) 1 + 0.25s
3a Write down the expression for the time response of the open loop system if m(t) is a unit step
voltage input applied at time t = 0. What is the steady state value of (t)?

-11-
EEET2197 Mechatronics & Control REVISION EXERCISE 1 (2012)

How long will it take for (t) to reach 98% of the steady state value? 6%
3b What additional components would you require to implement closed-loop speed control of the DC 12%
motor? Draw a block diagram of the closed loop system and mark each block and each signal in the
block diagram.
If a proportional controller (i.e., Gc(s) = K) is to be used, determine the gain K of the proportional
controller so that the settling time is reduced by a factor of 10. You may assume that the
tacho-generator constant is 1 V/(rad/s). What then is the steady state error for a unit step input?
3c What would be the benefit if an integral controller (i.e., Gc(s) = K/s) is implemented instead of the 2%
proportional controller?
3aS This is a first-order transfer function with a static gain of 60 (rad/s)/V, and time constant 4%
0.25seconds. Unit step response is, (t) 60(1-e-4t) rad/s
The steady state value of(t) = 60 rad/s 1%
For (t) to reach 98% of the steady state, time = 5 x time constant = 5x0.15=1.25 1%
3bS Tachogenerator, Reference signal generator, comparator, and controller 2%

4%
Controller gain to reduce the settling time by a factor of 10 is found as follows: 4%
60 60
(s )
= 1 + 0.25s = = 1 + 60K
60K
Closed-loop transfer function is,
R (s ) 60 x1 1 + 0.25s + 60K 0.25
1+ 1+ s
1 + 0.25s 1 + 60K
The new time constant is, 60K/[1+60K]
Hence, 1+60K = 10, or K = 0.15. The steady state error for a unit step input would be, 2%
1-Static gain =1-60K/[1+60K]=1/[1+60K]=1/10=0.1 or 10% of the reference Input
3cS In this case, the system becomes a TYPE 1 system, and the steady state error for a step input will be 2%
zero.
Q4 A closed-loop speed control system is shown in the Figure 4 below.

r(t) + e(t) Integral m(t) Voltage-Voltage va(t) PM DC m(t)


Controller Power Amplifier Motor
-

DC
Tacho-generator
vt(t)
Figure 4
The power amplifier has a gain of 2 volt/volt. That is, va (t ) = 2m(t ) volts
The tacho-generator constant is 0.02 volts/(rad/s). That is, vt (t ) = 0.02m (t ) volts
m (s ) 10
The transfer function of the PM DC motor, Gm (s ) =
Va (s ) 1 + 0.02 s
M (s ) 50
The transfer function of the integral controller, Gc (s ) = =
E (s ) s
4a If the motor is running at a steady speed of 200 rad/s, what are the corresponding (steady state) 5%
values of the following signals?
-12-
EEET2197 Mechatronics & Control REVISION EXERCISE 1 (2012)

Feedback signal, vt(t) Armature voltage, va(t)


Manipulated signal, m(t) Error signal, e(t) Reference signal, r(t)
4b Suppose the reference signal is then changed to -1, what then will be the new steady state values of 3%
the motor speed, armature voltage, and manipulated signal?
4c What is the transfer function of the closed-loop system? What are its poles? Determine the 12%
percentage overshoot and settling time for a step reference input.
4aS Feedback signal, vt(t) 0.02200 = 4 V 5%
Armature voltage, va(t) 200/10 = 20 V
Manipulated signal, m(t) 20/2 = 10 V
Error signal, e(t) = 0 V
Reference signal, r(t) = 4 V
4bS Motor Speed, (t) -1/0.02 = - 50 rad/s 3%
Armature voltage, va(t) -50/10 = -5 V
Manipulated signal, m(t) -5/2 = -2.5 V
4cS 50 10 12%
x2 x
m (s ) 1 + 0.02 s = 1000 50000
= s =
R (s ) 1 + 50 x 2 x 10 x0.02 s + 0.02 s 2 + 20 s 2 + 50 s + 1000
s 1 + 0.02 s
Poles are : -25 j(1000-625)=-25j19.4 Natural frequency, n=1010=31.6 rad/s
Damping ratio, = 50/(2x1010)=0.79
Percentage overshoot = 100 exp [-/(1-2)]=1.7% Settling time = 4/ 25 = 0.16 second
Q5 Consider the problem of maintaining the level of water in a tank such as the one shown in Figure 5
below. Water is drawn out at a variable rate, and the requirement is to supply a control amount of
water to maintain the level of water in the tank constant. Volume of water in the tank is A.h(t),
where A is the area of cross section of the tank. The rate of change of this volume equals the net
flow rate of water into the tank.
5a Develop the transfer function H(s)/Qin(s) for the 12%
following two cases:
(i) qout is independent of h, and determined qin
entirely by conditions external to the water tank
(i.e., qout is a disturbance input).
(ii) qout is dependent only on h as determined by
the equation, qout = R h, where R is a constant h
q out

Figure 5
5b For each case of the transfer function, H (s ) , mark the pole of the transfer function in the s-plane. 2%
Qin (s )
5c Suppose that qout remains constant, and that the water level in the tank has been adjusted to be equal 6%
to the desired height, hD. qin is then made equal to qout by manual adjustment of the inlet valve and
the system is left unattended for a long time. What do you think will happen in the two cases?
Explain why the system would respond quite differently in the two cases, and relate the two
contrasting responses to the s plane location of the pole of the transfer function.
5ai Differential equation of flow: 6%
S dh
qin (t ) qout (t ) = a
dt
Qin ( s ) Qout ( s ) = AsH ( s )
Laplace Transform:
Transfer function :
H (s) 1
= , Poles is at s=0
Qin ( s ) Qout ( s ) As

-13-
EEET2197 Mechatronics & Control REVISION EXERCISE 1 (2012)

5aii Qout is dependent only on h as determined by the 6%


S equation Qout = Rh, where R is a constant
Transfer function :
1 1
H (s) As
= = R
Qin ( s ) 1 A
1+ xR 1 + s , Poles at R/A
As R
5bS Respective poles are 0 and R/A

5cS Case (i) - Pole is at s = 0 (on the imaginary axis) which indicates a marginally stable system. In this 3%
case, the outcome is not predictable. It will be practically impossible to exactly match Qin to Qout,
with the result that, given enough time, the tank will either overflow or run dry depending on
whether Qin is higher or lower than Qout, however small their difference is.
Case (ii) - Pole is at s = -R/A (on the negative real axis) which indicates a stable system. In this case, 3%
any minor variation in Qin will result is a minor change in H(s), which in turn changes Qout to
exactly match Qin. Thus the height H(s) will remain practically constant as long as Qin remains very
nearly a constant.
Q6 Consider the feedback control system shown in Figure 6. The following lead controller is to
K ( s + 5)
be considered for the system. Gc (s ) =
( s + 40)
The design specifications are: Settling Time (Ts) < 0.2 seconds, and Percentage Overshoot
(PO) < 5% for a step input. It is also desirable to have as short a rise time as possible.

Figure 6.
6.a Sketch the root locus and comment of the effect of increasing the value of K on (i) the
stability and (ii) transient response to a step input of the closed-loop system.
12%
6.b What value of K would you recommend and why?
2%
6.c What then is the closed-loop transfer function, and what are its poles and zero?
5%
6.d Does the zero of the transfer function have any effect on the transient response? Explain your
1%
answer.
6aS K ( s + 5) 12%
Characterisitc Equation: 1 + Gc (s )G p (s ) = 1 + =0
s ( s + 5)( s + 40)
Rules for plotting the root locus diagram:
Locate open-loop poles (-p1,-p2,.. -pn) and zeros (-z1, -z2,.. -zm) in the s-plane (using symbols
for poles, and o for zeros)
Open-loop poles are 0, -5 and 40 (n=3)
Open loop zero is 5 (m=1)
Segment between 0 and -40 is part of the root loci
Determine the points at which the locus crosses the imaginary axis (if it does so), by substituting s =
j in the characteristic equation and by equating the real and imaginary parts separately to zero.
Not applicable in this problem (no feasible solution).

-14-
EEET2197 Mechatronics & Control REVISION EXERCISE 1 (2012)

Number of loci is n. Of these, m loci end at the m zeros. The


remaining n-m loci proceed to infinity along asymptotes cantered

at A and with angles A where A =


( p ) ( z )
j i
and
nm
2k + 1
A = 180 0 , k = 0,1,2,... .
nm
(0 5 40) (5)
A = = 20
3 1
2k + 1
A = 1800 = 90
3 1
Deteremine Break-away/break-in points (if any) by equating dK/ds=0
dK
K = s ( s + 40) = s 2 40 s = 2s 40 , Hence s = -20 is the breakaway point.
ds
Determine the angle of departure from a complex pole/angle of arrival at a complex zero by applying
the angle condition to a neighbouring point of the particular pole/zero.
Not applicable in this problem (no complex open-loop pole or zero).
Although the system is of second order, at values of K in the range 0 < K < 155.9, the system
can be approximated to a first order system with a pole at -20 + (360-K).The other pole will
be at more than six times away from the j-axis. In the range 155.9 < K < 400, the second order
system is overdamped with a sluggish step response.
The system is critically damped (=1) when K = 400. The two poles are identical & equal to -20.
The system has complex conjugate poles and damped oscillatory response for K > 400. Greater the
value K, smaller will be the damping ratio. However, the system will always be stable.
Settling time remains at 4/20 =0.2 second, irrespective of the value of K beyond 400.
6bS Value of K corresponding to the limiting damping ratio of = 0.5 should be chosen as a 2%
compromise between fast rise time, minimum overshoot and fast settling time.
The required value of K is given by, K = (20/0.5)2=1600
6cS K ( s + 5) 5%
s( s + 5)( s + 40) K ( s + 5) 1600( s + 5)
= =
K ( s + 5) s ( s + 5)( s + 40) + K ( s + 5) ( s + 5)( s 2 + 40s + 1600)
1+
s ( s + 5)( s + 40)
Zero: -5 Poles are : -5, -20 j203
6dS No, its effect is neutralized by the closed-loop pole at the same location (-5). 1%
EEET2197 DEC 2009 EXAM
EEET2197 DEC 2009 EXAM
Q1. Increasingly stringent requirements of modern high-precision machinery are placing increasing
demands on slide systems. The goal is to accurately control the desired path of the table shown in
Figure 1. The table is to be moved in the x direction as shown.
1.a What is a slide? What function does it perform? 2%
1.b What additional hardware components are required to
implement feedback control of the position x of the table, and
what functions do they perform? 6%
1.c Sketch a block diagram model of the feedback control system.
Label each block and each signal in the block diagram. 10%
1.d What is likely to be a disturbance input in this system? 2%

1aS A slide is a mechanical device that provides for accurate motion of a rigid body along a line while 2%
minimising friction and any lateral displacement.

-15-
EEET2197 Mechatronics & Control REVISION EXERCISE 1 (2012)

1bS Additional components/devices required: 6%


 Actuator (electric motor)
 Controlled power supply to motor (motor drive unit)
 Mechanical transmission to convert rotary motion of the motor to linear motion of the slide
 (lead-screw)
 Position sensor (usually a encoder attached to the motor shaft)
 Signal-processing electronics (analogue/digital hardware)
 Reference signal generator (usually a digital computer)
1cS 10%

1dS Disturbance input is the force exerted by the cutting tool and frictional forces. 2%
Q2. Using the Laplace Transform Table, find the inverse transforms (i.e., g(t)) of the following 8%
(a) Laplace Transform functions: (Hints: s24s+104=(s2)2+102)
s+2 10
(i) G ( s) = 2 (ii) G ( s) = 2
s + 4 s + 104 s 4 s + 104
(b) Draw rough sketches (graphs) to show how g(t) varies with time t in each case in the time 8%
interval 0 t 2 x 2 /10 seconds. What is significant about this time interval?
(c) Comment on how their magnitudes change with the passage of time. 4%
2aS s+2 ( s + 2) 4%
(i) G ( s) = s 2 + 4 s + 104 = ( s + 2) 2 + 10 2 g (t ) = e 2t cos(10t )

(ii) 10 10 4%
G ( s) = 2 = g (t ) = e 2t sin(10t )
s 4 s + 104 ( s 2) + 102 2

2b 2b The response in each case is Fig.


S oscillatory, and 2second is the 6%
period of the oscillations. Thus, the
function g(t) undergoes two 2%
complete oscillations in 2x2
seconds

2c The first function is a cosine 4%


function with exponentially
decreasing magnitude; the second
function is a sine function with
exponentially increasing
magnitude.

Q3 A system is shown in Figure 3(a). Determine G(s) and H(s) of the block diagram shown in 10%
(a) Figure 3(b) that is equivalent to the block diagram of Figure 3(a). Determine Y(s)/R(s).

(a) (b) Figure 3a


-16-
EEET2197 Mechatronics & Control REVISION EXERCISE 1 (2012)

(b) A mechanical system is depicted in Figure 3b. The input 10%


signal is the force f(t) directly applied to the mass m, and the
output signal is the vertical displacement of the mass.
Suppose m = 2.5 kg, k = 200 N/m, and b = 16 N/(m/s).
Determine the transfer function Y(s)/F(s).

Figure 3b
3aS An equivalent block diagram may be drawn as follows:

1 10%
Hence, the forward path transfer function, G ( s ) =
( s + 5)( s + 10)
, and the feedback transfer function, H(s)=(s+10-s-5)=5
1
( s + 5)( s + 10) 1 1
Hence, the transfer function is GCL ( s) = = = 2
1x5 ( s + 5)( s + 10) + 5 s + 15s + 55
1+
( s + 5)( s + 10)

-17-
EEET2197 Mechatronics & Control REVISION EXERCISE 1 (2012)

3bS

Applying Newtons Law to the mass 10%


d2y dy
m 2 = f (t ) ky b
dt dt
ms Y ( s) = F ( s ) kY ( s ) bsY ( s ) ms 2Y ( s ) + bsY ( s ) + kY ( s ) = F ( s)
2

Y ( s) 1 1
Hence, G ( s) = = =
F ( s) ms + bs + k 2.5s + 16s + 200
2 2

Q4 Express the following transfer function in the standard form and hence determine its 6%
(a) characteristic parameters (static gain k, natural frequency n and damping ratio):
3000
T (s) =
0.75s + 42 s + 1200
2

Sketch the response of the system (i.e. the output of the system) for a unit step input, and mark the
key features of the response (such as the steady state value, settling time etc). 6%
(b) 0.3( s + 50)
A control system has the following transfer function: T ( s ) =
( s + 0.1)( s 2 + 6 s + 25)
Show that the above transfer function can be approximated by a first order transfer function of
k
the form, T ( s ) 6%
1 + s
Determine the two characteristic parameters of the approximated transfer function, namely, the 2%
static gain k and the time constant .
4aS

6%

6%
(Fig
Static gain, k=2.5; Natural frequency, n =40 rad/s; Damping ration, =0.7 .)
4bS 6%

Static gain, k = 6; Time constant, = 10 s 2%


-18-
EEET2197 Mechatronics & Control REVISION EXERCISE 1 (2012)

Q5 The speed control of a high-speed train is represented by the block diagram of Figure 5.

Figure 5
Determine the following:
5.a the transfer functions Y(s) / R(s) and Y(s) / Td(s) 4%
5.b steady-state error for a unit step input 4%
5.c closed-loop poles, their damping ratio and natural frequency 6%
5.d percentage overshoot and settling time (2% criterion) for a step input. 6%
5aS The closed-loop transfer function, 2%
Y (s) 20 K 200
G R ( s) = = 2 = 2
R ( s ) s + 14s + 48 + 20 K s + 14s + 248
The closed-loop transfer function, 2%
Y (s) 20 20
GTd ( s ) = = 2 = 2
Td ( s ) s + 14 s + 48 + 20 K s + 14 s + 248
5bS 20 K 4%
Forward path transfer function of the unity feedback system, G ( s) = 2
s + 14 s + 48
This is a TYPE 0 system, the position error constant Kp = G(0) = 20K / 48
1 1 1
Steady-state error for a unit step input, ess = = = = 0.19
1 + K p 1 + 20 K / 48 1 + 5 K / 12
5cS 14 196 4 x(48 + 20 K ) 2%
Closed-loop poles are : s = = 7 1 20 K = 7 j14.1
2

The natural frequency is : n = 48 + 20K = 15.75rad / s 2%


14 7
And the damping ratio is : = = = 0.44 2%
2 48 + 20 K 48 + 20 K
5dS 7
Percentage overshoot for a step input is PO = 100 exp = 100 exp = 21.04% 3%
1
2
20 K 1
And the settling time (2% criterion) for a step input is : Ts = 4 / [ n] = 4/7 seconds 3%

Q6. A tape recorder has a speed control system with negative feedback and H(s) = 1. The forward path
K
transfer function is G ( s) =
s ( s + 2)( s + 4)( s + 6)
6.a Show that the break-away points of the root locus are given by two of the three roots -5.24,
-3.00 and -0.76 of the cubic equation 4s3+36s2+88s+48 Which of these are the two 4%
break-away points?
6.b Sketch the root locus diagram on the given graph paper. 10%
6.c By applying the angle condition, show that -0.664 + j0.677 is a point on the root locus. By 6%
applying the magnitude condition to this point, determine the corresponding value of K.
6aS K
The characteristic equation is 1 + =0
s ( s + 2)( s + 4)( s + 6)
Number of Open-loop zeros: None Number of Open-loop poles: 4
Open-loop poles: 0, -2, -4 and -6
Locus on the real axis: Segments between 0 and -2, and between -4 and -6 are part of the root loci.
There will be a breakaway point in each of these segments.
-19-
EEET2197 Mechatronics & Control REVISION EXERCISE 1 (2012)

Break-away points are determined as follows:


dK
K = s( s + 2)( s + 4)( s + 6) = ( s 4 + 12 s 3 + 44 s 2 + 48s ) = (4 s 3 + 36 s 2 + 88s + 48) = 0
ds
The three roots are -5.24, -3.00 and -0.76. Since the break-away points are expected to be in the
intervals (0, -2) and (-4, -6), the two breakaway points are -0.76 and -5.24. 4%
Asymptotes
( 0 2 4 6)
Meeting point of the asymptotes on the x-axis, A = = 3
40
180 k 360 2%
Angles of the asymptotes A = =45o or135o
4
Crossing of the j-axis: Characteristic equation is s 4 + 12 s 3 + 44 s 2 + 48s + K = 0
Substituting s=j 4 j12 3 44 2 + j 48 + K = 0
Equating the real and imaginary parts separately to zero:
4 44 2 + K = 0 j12 3 + j 48 = 0 Hence, 2 + 4 = 0 or =2 rad/s and K=160 2%
6bS

6cS 1 6%
If s = -0.664+j0.677, angle of =0 [134.44+26.87+11.47+7.23]= -180o. Thus
s( s + 2)( s + 4)( s + 6)
the angle condition is satisfied. Applying the magnitude condition to the 0.664+j0.677
K= 1 / s ( s + 2)( s + 4)( s + 6) s = 0.664+ j 0.667 = 26.00

-20-

You might also like